Difference between revisions of "2003 AMC 10B Problems/Problem 1"

m
(Redirected page to 2003 AMC 12B Problems/Problem 1)
 
(4 intermediate revisions by 3 users not shown)
Line 1: Line 1:
==Problem==
+
#REDIRECT [[2003 AMC 12B Problems/Problem 1]]
 
 
Which of the following is the same as <math>\dfrac{2-4+6-8+10-12+14}{3-6+9-12+15-18+21}</math>?
 
 
 
<math>\textbf{(A)}\ -1 \qquad
 
\textbf{(B)}\ -\frac23 \qquad
 
\textbf{(C)}\ \frac23 \qquad
 
\textbf{(D)}\ 1 \qquad
 
\textbf{(E)}\ \frac{14}{3}</math>
 
 
 
==Solution==
 
 
 
{{solution}}
 
 
 
==See Also==
 
 
 
{{AMC10 box|year=2003|ab=B|before=First Question|num-a=2}}
 
[[Category: Introductory Algebra Problems]]
 

Latest revision as of 00:02, 5 January 2014